2022 AMC 10B Problems/Problem 23

Revision as of 15:11, 17 November 2022 by Stevenyiweichen (talk | contribs) (Solution)

Solution

We use the following lemma to solve this problem.


Let $y_1, y_2, \cdots, y_n$ be independent random variables that are uniformly distributed on $(0,1)$. Then for $n = 2$, \[ \Bbb P \left( y_1 + y_2 \leq 1 \right) = \frac{1}{2} . \]

For $n = 3$, \[ \Bbb P \left( y_1 + y_2 + y_3 \leq 1 \right) = \frac{1}{6} . \]


Now, we solve this problem.

We denote by $\tau$ the last step Amelia moves. Thus, $\tau \in \left\{ 2, 3 \right\}$. We have

\begin{align*}
P \left( \sum_{n=1}^\tau x_n > 1 \right)
& = P \left( x_1 + x_2 > 1 | t_1 + t_2 > 1 \right)
 P \left( t_1 + t_2 > 1 \right) \\
& \hspace{1cm} + P \left( x_1 + x_2 + x_3 > 1 | t_1 + t_2 \leq 1 \right)
P \left( t_1 + t_2 \leq 1 \right) \\
& = P \left( x_1 + x_2 > 1 \right)
P \left( t_1 + t_2 > 1 \right)
+ P \left( x_1 + x_2 + x_3 > 1 \right)
P \left( t_1 + t_2 \leq 1 \right) \\
& = \left( 1 - \frac{1}{2} \right)\left( 1 - \frac{1}{2} \right)
+ \left( 1 - \frac{1}{6} \right) \frac{1}{2} \\
& = \boxed{\textbf{(C) \frac{2}{3}}} ,
\end{align*} (Error compiling LaTeX. Unknown error_msg)

where the second equation follows from the property that $\left\{ x_n \right\}$ and $\left\{ t_n \right\}$ are independent sequences, the third equality follows from the lemma above.

~Steven Chen (Professor Chen Education Palace, www.professorchenedu.com)

Video Solution

https://youtu.be/qOxnx_c9kVo

~Steven Chen (Professor Chen Education Palace, www.professorchenedu.com)